Prove that $f=1/sqrt{x}$ is continuous on the interval $(0,1]$, but not uniformly continuous.












3












$begingroup$


Prove that $f(x)=1/sqrt{x}$ is continuous on the interval $(0,1]$, but not uniformly continuous.



I believe it follows that $f(x)$ is not uniformly continuous because $f(x)$ is not continuous on the bounded interval including zero. I tried to set up an $epsilon-delta$ proof of the continuity on $(0,1]$ with $delta=frac{p}{(2epsilon+1)^2}$ where $p$ is any point in the interval, but I feel like I am making a leap to say that by that $delta$ we get $|frac{sqrt p-sqrt x}{sqrt psqrt x}|ltepsilon$










share|cite|improve this question









$endgroup$








  • 2




    $begingroup$
    Actually, it's because the slope of the tangent line approaches infinity as $xto 0$. Continuous means there is a $delta$ such that blah blah blah, where $delta$ can depend on $x$; uniform continuity means that $delta$ does not depend on $x$.
    $endgroup$
    – Christopher Carl Heckman
    Feb 29 '16 at 1:30






  • 1




    $begingroup$
    Notice that a uniformly continuous function maps Cauchy sequences into Cauchy sequences. In other words, you may disprove the uniform continuity of $f$ by choosing a suitable Cauchy sequence in $(0, 1]$ which is not mapped to a Cauchy sequence.
    $endgroup$
    – Sangchul Lee
    Feb 29 '16 at 1:39










  • $begingroup$
    But can't you also show that, once you have proved continuity on $(0,1]$, that the function is not continuous on the closed interval $[0,1]$, and thus it can not be uniformly continuous on the same interval?
    $endgroup$
    – e2DAeyePi
    Feb 29 '16 at 4:08
















3












$begingroup$


Prove that $f(x)=1/sqrt{x}$ is continuous on the interval $(0,1]$, but not uniformly continuous.



I believe it follows that $f(x)$ is not uniformly continuous because $f(x)$ is not continuous on the bounded interval including zero. I tried to set up an $epsilon-delta$ proof of the continuity on $(0,1]$ with $delta=frac{p}{(2epsilon+1)^2}$ where $p$ is any point in the interval, but I feel like I am making a leap to say that by that $delta$ we get $|frac{sqrt p-sqrt x}{sqrt psqrt x}|ltepsilon$










share|cite|improve this question









$endgroup$








  • 2




    $begingroup$
    Actually, it's because the slope of the tangent line approaches infinity as $xto 0$. Continuous means there is a $delta$ such that blah blah blah, where $delta$ can depend on $x$; uniform continuity means that $delta$ does not depend on $x$.
    $endgroup$
    – Christopher Carl Heckman
    Feb 29 '16 at 1:30






  • 1




    $begingroup$
    Notice that a uniformly continuous function maps Cauchy sequences into Cauchy sequences. In other words, you may disprove the uniform continuity of $f$ by choosing a suitable Cauchy sequence in $(0, 1]$ which is not mapped to a Cauchy sequence.
    $endgroup$
    – Sangchul Lee
    Feb 29 '16 at 1:39










  • $begingroup$
    But can't you also show that, once you have proved continuity on $(0,1]$, that the function is not continuous on the closed interval $[0,1]$, and thus it can not be uniformly continuous on the same interval?
    $endgroup$
    – e2DAeyePi
    Feb 29 '16 at 4:08














3












3








3





$begingroup$


Prove that $f(x)=1/sqrt{x}$ is continuous on the interval $(0,1]$, but not uniformly continuous.



I believe it follows that $f(x)$ is not uniformly continuous because $f(x)$ is not continuous on the bounded interval including zero. I tried to set up an $epsilon-delta$ proof of the continuity on $(0,1]$ with $delta=frac{p}{(2epsilon+1)^2}$ where $p$ is any point in the interval, but I feel like I am making a leap to say that by that $delta$ we get $|frac{sqrt p-sqrt x}{sqrt psqrt x}|ltepsilon$










share|cite|improve this question









$endgroup$




Prove that $f(x)=1/sqrt{x}$ is continuous on the interval $(0,1]$, but not uniformly continuous.



I believe it follows that $f(x)$ is not uniformly continuous because $f(x)$ is not continuous on the bounded interval including zero. I tried to set up an $epsilon-delta$ proof of the continuity on $(0,1]$ with $delta=frac{p}{(2epsilon+1)^2}$ where $p$ is any point in the interval, but I feel like I am making a leap to say that by that $delta$ we get $|frac{sqrt p-sqrt x}{sqrt psqrt x}|ltepsilon$







real-analysis continuity






share|cite|improve this question













share|cite|improve this question











share|cite|improve this question




share|cite|improve this question










asked Feb 29 '16 at 1:23









e2DAeyePie2DAeyePi

124110




124110








  • 2




    $begingroup$
    Actually, it's because the slope of the tangent line approaches infinity as $xto 0$. Continuous means there is a $delta$ such that blah blah blah, where $delta$ can depend on $x$; uniform continuity means that $delta$ does not depend on $x$.
    $endgroup$
    – Christopher Carl Heckman
    Feb 29 '16 at 1:30






  • 1




    $begingroup$
    Notice that a uniformly continuous function maps Cauchy sequences into Cauchy sequences. In other words, you may disprove the uniform continuity of $f$ by choosing a suitable Cauchy sequence in $(0, 1]$ which is not mapped to a Cauchy sequence.
    $endgroup$
    – Sangchul Lee
    Feb 29 '16 at 1:39










  • $begingroup$
    But can't you also show that, once you have proved continuity on $(0,1]$, that the function is not continuous on the closed interval $[0,1]$, and thus it can not be uniformly continuous on the same interval?
    $endgroup$
    – e2DAeyePi
    Feb 29 '16 at 4:08














  • 2




    $begingroup$
    Actually, it's because the slope of the tangent line approaches infinity as $xto 0$. Continuous means there is a $delta$ such that blah blah blah, where $delta$ can depend on $x$; uniform continuity means that $delta$ does not depend on $x$.
    $endgroup$
    – Christopher Carl Heckman
    Feb 29 '16 at 1:30






  • 1




    $begingroup$
    Notice that a uniformly continuous function maps Cauchy sequences into Cauchy sequences. In other words, you may disprove the uniform continuity of $f$ by choosing a suitable Cauchy sequence in $(0, 1]$ which is not mapped to a Cauchy sequence.
    $endgroup$
    – Sangchul Lee
    Feb 29 '16 at 1:39










  • $begingroup$
    But can't you also show that, once you have proved continuity on $(0,1]$, that the function is not continuous on the closed interval $[0,1]$, and thus it can not be uniformly continuous on the same interval?
    $endgroup$
    – e2DAeyePi
    Feb 29 '16 at 4:08








2




2




$begingroup$
Actually, it's because the slope of the tangent line approaches infinity as $xto 0$. Continuous means there is a $delta$ such that blah blah blah, where $delta$ can depend on $x$; uniform continuity means that $delta$ does not depend on $x$.
$endgroup$
– Christopher Carl Heckman
Feb 29 '16 at 1:30




$begingroup$
Actually, it's because the slope of the tangent line approaches infinity as $xto 0$. Continuous means there is a $delta$ such that blah blah blah, where $delta$ can depend on $x$; uniform continuity means that $delta$ does not depend on $x$.
$endgroup$
– Christopher Carl Heckman
Feb 29 '16 at 1:30




1




1




$begingroup$
Notice that a uniformly continuous function maps Cauchy sequences into Cauchy sequences. In other words, you may disprove the uniform continuity of $f$ by choosing a suitable Cauchy sequence in $(0, 1]$ which is not mapped to a Cauchy sequence.
$endgroup$
– Sangchul Lee
Feb 29 '16 at 1:39




$begingroup$
Notice that a uniformly continuous function maps Cauchy sequences into Cauchy sequences. In other words, you may disprove the uniform continuity of $f$ by choosing a suitable Cauchy sequence in $(0, 1]$ which is not mapped to a Cauchy sequence.
$endgroup$
– Sangchul Lee
Feb 29 '16 at 1:39












$begingroup$
But can't you also show that, once you have proved continuity on $(0,1]$, that the function is not continuous on the closed interval $[0,1]$, and thus it can not be uniformly continuous on the same interval?
$endgroup$
– e2DAeyePi
Feb 29 '16 at 4:08




$begingroup$
But can't you also show that, once you have proved continuity on $(0,1]$, that the function is not continuous on the closed interval $[0,1]$, and thus it can not be uniformly continuous on the same interval?
$endgroup$
– e2DAeyePi
Feb 29 '16 at 4:08










2 Answers
2






active

oldest

votes


















0












$begingroup$

Recall that $f$ is uniformly continuous if and only if for each $varepsilon>0$, there exists $delta>0$ such that
$$sup{|f(x)-f(y)| : |x-y|<delta}<varepsilon.$$
Consider the sequence $x_n=frac1{n^2}$. Then for any $N$,



$$
|x_N-x_{N+m}| = frac1{N^2}-frac1{(N+m)^2} stackrel{mtoinfty}longrightarrowfrac1{N^2},
$$
and
$$|f(x_N)-f(x_{N+m})|=left|frac1N-frac1{N+m} right|=frac{m}{N(N+m)}. $$
Choose $varepsilon=frac12$. Then for any $delta>0$, we may choose $N$ such that $frac1{N^2}<delta$, but
$$f(x_N)-f(x_{N+m}) = frac{m}{N(N+m)}> frac m{delta+m}stackrel{mtoinfty}longrightarrow 1,$$
so for $m$ sufficiently large, $$|f(x_N)-f(x_{N+m})|>frac12.$$ It follows that $f$ is not uniformly continuous






share|cite|improve this answer









$endgroup$





















    0












    $begingroup$

    Here we can use the following sequential criterion:




    If $f$ is not uniformly continuous on a given domain $D$, then we can find two sequences $(x_n)$ and $(y_n)$ in $D$ such that $lim_{ntoinfty}(x_n-y_n)=0$ and $|f(x_n)-f(y_n)|geepsilon_0$ for some fixed positive $epsilon_0.$




    Simply choose $x_n=dfrac{1}{n^2}$ and $y_n=dfrac{1}{4n^2}$ for all $ninmathbb{N}.$






    share|cite|improve this answer











    $endgroup$













    • $begingroup$
      One would need the reverse implication.
      $endgroup$
      – Did
      Apr 27 '16 at 8:19












    Your Answer








    StackExchange.ready(function() {
    var channelOptions = {
    tags: "".split(" "),
    id: "69"
    };
    initTagRenderer("".split(" "), "".split(" "), channelOptions);

    StackExchange.using("externalEditor", function() {
    // Have to fire editor after snippets, if snippets enabled
    if (StackExchange.settings.snippets.snippetsEnabled) {
    StackExchange.using("snippets", function() {
    createEditor();
    });
    }
    else {
    createEditor();
    }
    });

    function createEditor() {
    StackExchange.prepareEditor({
    heartbeatType: 'answer',
    autoActivateHeartbeat: false,
    convertImagesToLinks: true,
    noModals: true,
    showLowRepImageUploadWarning: true,
    reputationToPostImages: 10,
    bindNavPrevention: true,
    postfix: "",
    imageUploader: {
    brandingHtml: "Powered by u003ca class="icon-imgur-white" href="https://imgur.com/"u003eu003c/au003e",
    contentPolicyHtml: "User contributions licensed under u003ca href="https://creativecommons.org/licenses/by-sa/3.0/"u003ecc by-sa 3.0 with attribution requiredu003c/au003e u003ca href="https://stackoverflow.com/legal/content-policy"u003e(content policy)u003c/au003e",
    allowUrls: true
    },
    noCode: true, onDemand: true,
    discardSelector: ".discard-answer"
    ,immediatelyShowMarkdownHelp:true
    });


    }
    });














    draft saved

    draft discarded


















    StackExchange.ready(
    function () {
    StackExchange.openid.initPostLogin('.new-post-login', 'https%3a%2f%2fmath.stackexchange.com%2fquestions%2f1676501%2fprove-that-f-1-sqrtx-is-continuous-on-the-interval-0-1-but-not-uniform%23new-answer', 'question_page');
    }
    );

    Post as a guest















    Required, but never shown

























    2 Answers
    2






    active

    oldest

    votes








    2 Answers
    2






    active

    oldest

    votes









    active

    oldest

    votes






    active

    oldest

    votes









    0












    $begingroup$

    Recall that $f$ is uniformly continuous if and only if for each $varepsilon>0$, there exists $delta>0$ such that
    $$sup{|f(x)-f(y)| : |x-y|<delta}<varepsilon.$$
    Consider the sequence $x_n=frac1{n^2}$. Then for any $N$,



    $$
    |x_N-x_{N+m}| = frac1{N^2}-frac1{(N+m)^2} stackrel{mtoinfty}longrightarrowfrac1{N^2},
    $$
    and
    $$|f(x_N)-f(x_{N+m})|=left|frac1N-frac1{N+m} right|=frac{m}{N(N+m)}. $$
    Choose $varepsilon=frac12$. Then for any $delta>0$, we may choose $N$ such that $frac1{N^2}<delta$, but
    $$f(x_N)-f(x_{N+m}) = frac{m}{N(N+m)}> frac m{delta+m}stackrel{mtoinfty}longrightarrow 1,$$
    so for $m$ sufficiently large, $$|f(x_N)-f(x_{N+m})|>frac12.$$ It follows that $f$ is not uniformly continuous






    share|cite|improve this answer









    $endgroup$


















      0












      $begingroup$

      Recall that $f$ is uniformly continuous if and only if for each $varepsilon>0$, there exists $delta>0$ such that
      $$sup{|f(x)-f(y)| : |x-y|<delta}<varepsilon.$$
      Consider the sequence $x_n=frac1{n^2}$. Then for any $N$,



      $$
      |x_N-x_{N+m}| = frac1{N^2}-frac1{(N+m)^2} stackrel{mtoinfty}longrightarrowfrac1{N^2},
      $$
      and
      $$|f(x_N)-f(x_{N+m})|=left|frac1N-frac1{N+m} right|=frac{m}{N(N+m)}. $$
      Choose $varepsilon=frac12$. Then for any $delta>0$, we may choose $N$ such that $frac1{N^2}<delta$, but
      $$f(x_N)-f(x_{N+m}) = frac{m}{N(N+m)}> frac m{delta+m}stackrel{mtoinfty}longrightarrow 1,$$
      so for $m$ sufficiently large, $$|f(x_N)-f(x_{N+m})|>frac12.$$ It follows that $f$ is not uniformly continuous






      share|cite|improve this answer









      $endgroup$
















        0












        0








        0





        $begingroup$

        Recall that $f$ is uniformly continuous if and only if for each $varepsilon>0$, there exists $delta>0$ such that
        $$sup{|f(x)-f(y)| : |x-y|<delta}<varepsilon.$$
        Consider the sequence $x_n=frac1{n^2}$. Then for any $N$,



        $$
        |x_N-x_{N+m}| = frac1{N^2}-frac1{(N+m)^2} stackrel{mtoinfty}longrightarrowfrac1{N^2},
        $$
        and
        $$|f(x_N)-f(x_{N+m})|=left|frac1N-frac1{N+m} right|=frac{m}{N(N+m)}. $$
        Choose $varepsilon=frac12$. Then for any $delta>0$, we may choose $N$ such that $frac1{N^2}<delta$, but
        $$f(x_N)-f(x_{N+m}) = frac{m}{N(N+m)}> frac m{delta+m}stackrel{mtoinfty}longrightarrow 1,$$
        so for $m$ sufficiently large, $$|f(x_N)-f(x_{N+m})|>frac12.$$ It follows that $f$ is not uniformly continuous






        share|cite|improve this answer









        $endgroup$



        Recall that $f$ is uniformly continuous if and only if for each $varepsilon>0$, there exists $delta>0$ such that
        $$sup{|f(x)-f(y)| : |x-y|<delta}<varepsilon.$$
        Consider the sequence $x_n=frac1{n^2}$. Then for any $N$,



        $$
        |x_N-x_{N+m}| = frac1{N^2}-frac1{(N+m)^2} stackrel{mtoinfty}longrightarrowfrac1{N^2},
        $$
        and
        $$|f(x_N)-f(x_{N+m})|=left|frac1N-frac1{N+m} right|=frac{m}{N(N+m)}. $$
        Choose $varepsilon=frac12$. Then for any $delta>0$, we may choose $N$ such that $frac1{N^2}<delta$, but
        $$f(x_N)-f(x_{N+m}) = frac{m}{N(N+m)}> frac m{delta+m}stackrel{mtoinfty}longrightarrow 1,$$
        so for $m$ sufficiently large, $$|f(x_N)-f(x_{N+m})|>frac12.$$ It follows that $f$ is not uniformly continuous







        share|cite|improve this answer












        share|cite|improve this answer



        share|cite|improve this answer










        answered Feb 29 '16 at 2:10









        Math1000Math1000

        19.4k31746




        19.4k31746























            0












            $begingroup$

            Here we can use the following sequential criterion:




            If $f$ is not uniformly continuous on a given domain $D$, then we can find two sequences $(x_n)$ and $(y_n)$ in $D$ such that $lim_{ntoinfty}(x_n-y_n)=0$ and $|f(x_n)-f(y_n)|geepsilon_0$ for some fixed positive $epsilon_0.$




            Simply choose $x_n=dfrac{1}{n^2}$ and $y_n=dfrac{1}{4n^2}$ for all $ninmathbb{N}.$






            share|cite|improve this answer











            $endgroup$













            • $begingroup$
              One would need the reverse implication.
              $endgroup$
              – Did
              Apr 27 '16 at 8:19
















            0












            $begingroup$

            Here we can use the following sequential criterion:




            If $f$ is not uniformly continuous on a given domain $D$, then we can find two sequences $(x_n)$ and $(y_n)$ in $D$ such that $lim_{ntoinfty}(x_n-y_n)=0$ and $|f(x_n)-f(y_n)|geepsilon_0$ for some fixed positive $epsilon_0.$




            Simply choose $x_n=dfrac{1}{n^2}$ and $y_n=dfrac{1}{4n^2}$ for all $ninmathbb{N}.$






            share|cite|improve this answer











            $endgroup$













            • $begingroup$
              One would need the reverse implication.
              $endgroup$
              – Did
              Apr 27 '16 at 8:19














            0












            0








            0





            $begingroup$

            Here we can use the following sequential criterion:




            If $f$ is not uniformly continuous on a given domain $D$, then we can find two sequences $(x_n)$ and $(y_n)$ in $D$ such that $lim_{ntoinfty}(x_n-y_n)=0$ and $|f(x_n)-f(y_n)|geepsilon_0$ for some fixed positive $epsilon_0.$




            Simply choose $x_n=dfrac{1}{n^2}$ and $y_n=dfrac{1}{4n^2}$ for all $ninmathbb{N}.$






            share|cite|improve this answer











            $endgroup$



            Here we can use the following sequential criterion:




            If $f$ is not uniformly continuous on a given domain $D$, then we can find two sequences $(x_n)$ and $(y_n)$ in $D$ such that $lim_{ntoinfty}(x_n-y_n)=0$ and $|f(x_n)-f(y_n)|geepsilon_0$ for some fixed positive $epsilon_0.$




            Simply choose $x_n=dfrac{1}{n^2}$ and $y_n=dfrac{1}{4n^2}$ for all $ninmathbb{N}.$







            share|cite|improve this answer














            share|cite|improve this answer



            share|cite|improve this answer








            edited Apr 27 '16 at 10:52

























            answered Apr 27 '16 at 7:52









            BumblebeeBumblebee

            9,74912551




            9,74912551












            • $begingroup$
              One would need the reverse implication.
              $endgroup$
              – Did
              Apr 27 '16 at 8:19


















            • $begingroup$
              One would need the reverse implication.
              $endgroup$
              – Did
              Apr 27 '16 at 8:19
















            $begingroup$
            One would need the reverse implication.
            $endgroup$
            – Did
            Apr 27 '16 at 8:19




            $begingroup$
            One would need the reverse implication.
            $endgroup$
            – Did
            Apr 27 '16 at 8:19


















            draft saved

            draft discarded




















































            Thanks for contributing an answer to Mathematics Stack Exchange!


            • Please be sure to answer the question. Provide details and share your research!

            But avoid



            • Asking for help, clarification, or responding to other answers.

            • Making statements based on opinion; back them up with references or personal experience.


            Use MathJax to format equations. MathJax reference.


            To learn more, see our tips on writing great answers.




            draft saved


            draft discarded














            StackExchange.ready(
            function () {
            StackExchange.openid.initPostLogin('.new-post-login', 'https%3a%2f%2fmath.stackexchange.com%2fquestions%2f1676501%2fprove-that-f-1-sqrtx-is-continuous-on-the-interval-0-1-but-not-uniform%23new-answer', 'question_page');
            }
            );

            Post as a guest















            Required, but never shown





















































            Required, but never shown














            Required, but never shown












            Required, but never shown







            Required, but never shown

































            Required, but never shown














            Required, but never shown












            Required, but never shown







            Required, but never shown







            Popular posts from this blog

            Plaza Victoria

            In PowerPoint, is there a keyboard shortcut for bulleted / numbered list?

            How to put 3 figures in Latex with 2 figures side by side and 1 below these side by side images but in...